Proof of ##M^n## (matrix multiplication problem)

In summary: In other words, the order in the proof does not matter.Homework Statement: Please see belowRelevant Equations: Please see below
  • #1
ChiralSuperfields
1,276
134
Homework Statement
Please see below
Relevant Equations
Please see below
For,
1683666942440.png

Does anybody please know why they did not change the order in the second line of the proof? For example, why did they not rearrange the order to be ##M^n = (DP^{-1}P)(DP^{-1}P)(DP^{-1}P)(DP^{-1}P)---(DP^{-1}P)## for to get ##M^n = (DI)(DI)(DI)(DI)---(DI) = D^n##

Many thanks!
 
Physics news on Phys.org
  • #2
ChiralSuperfields said:
Homework Statement: Please see below
Relevant Equations: Please see below

For,
View attachment 326261
Does anybody please know why they did not change the order in the second line of the proof? For example, why did they not rearrange the order to be ##M^n = (DP^{-1}P)(DP^{-1}P)(DP^{-1}P)(DP^{-1}P)---(DP^{-1}P)## for to get ##M^n = (DI)(DI)(DI)(DI)---(DI) = D^n##

Many thanks!
They did exactly what you proposed only with one more step that shows how the associativity law is necessary here. We first need ##(AB)B^{-1}=A(BB^{-1})=AI=A## before we are allowed to write ##A##. It is important to recognize which laws of matrix multiplication are actually used. Here it's the law of associativity ##(AB)B^{-1}=A(BB^{-1})##, the definition of an inverse ##BB^{-1}=I## and the definition of the neutral element ##AI=A.## So every single property of matrix multiplication, except for the distributive law since there is no addition here, has actually been used.
 
  • Like
Likes ChiralSuperfields
  • #3
ChiralSuperfields said:
Homework Statement: Please see below
Relevant Equations: Please see below

For,
View attachment 326261
Does anybody please know why they did not change the order in the second line of the proof? For example, why did they not rearrange the order to be ##M^n = (DP^{-1}P)(DP^{-1}P)(DP^{-1}P)(DP^{-1}P)---(DP^{-1}P)## for to get ##M^n = (DI)(DI)(DI)(DI)---(DI) = D^n##

Many thanks!
Because that would not be correct. Matrix multiplication is not commutative, and it is definitely not true that a matrix, ##M^n## that is probably non-diagonal, is equal to the diagonal matrix ##D^n##.
 
  • Like
Likes Infrared, ChiralSuperfields, vela and 1 other person
  • #4
@FactChecker has a good point here. I didn't see that you switched the order of ##P## and ##D.##

The only matrices that can be swapped are the diagonal matrices with the same value on the entire diagonal.
$$
A\cdot B = B\cdot A \text{ for all matrices } A \Longleftrightarrow D=\operatorname{diag}(d,d,\ldots,d)
$$

So if we do not have any specific information about ##A,## we must treat it like an arbitrary matrix. And that leaves us with ##\begin{pmatrix}d&0&\ldots&0\\0&d&\ldots&0\\ \vdots&\vdots&\ddots&\vdots\\ 0&0&\ldots&d\\ \end{pmatrix}## as the only matrix that commutes with ##A.## If we consider a specific matrix ##A,## then there are possibly more matrices that commute with ##A##. (Commute means ##A\cdot B=B\cdot A.##) However, if all these matrices are as before, then ##PD\neq DP.##
 
  • Like
Likes ChiralSuperfields

Similar threads

  • Calculus and Beyond Homework Help
Replies
3
Views
404
  • Calculus and Beyond Homework Help
Replies
24
Views
898
  • Calculus and Beyond Homework Help
Replies
1
Views
566
  • Calculus and Beyond Homework Help
Replies
4
Views
872
  • Calculus and Beyond Homework Help
Replies
2
Views
159
  • Calculus and Beyond Homework Help
Replies
8
Views
1K
  • Calculus and Beyond Homework Help
Replies
3
Views
555
  • Calculus and Beyond Homework Help
Replies
7
Views
396
  • Calculus and Beyond Homework Help
Replies
5
Views
725
  • Calculus and Beyond Homework Help
Replies
17
Views
826
Back
Top